From Cartesian to spherical integral

Click For Summary
The discussion revolves around setting up a spherical integral for a homework problem. The integral setup initially presented includes limits for theta and phi, with some confusion regarding the limits for rho. Participants clarify that while rho starts from 0, it should extend to the original limit of sqrt(2) rather than 1. There is also a suggestion to simplify the integral by canceling out rho^2 factors. The conversation highlights the challenges of transitioning from Cartesian to spherical coordinates in integral calculus.
Feodalherren
Messages
604
Reaction score
6

Homework Statement


coords.png



Homework Equations





The Attempt at a Solution


Is this the correct setup?

\int^{\pi}_{\frac{3\pi}{4}}\int^{2\pi}_{0}\int^{\sqrt{2}}_{0}\frac{1}{\rho^{2}} rho^2 Sin\phi d\rho d\theta d\phi

I gave up on itex. It was either that or my computer flying out the window. My apologies.
 
Last edited:
Physics news on Phys.org
You came up with:
$$\int_{3\pi/4}^\pi \int_0^{2\pi}\int_0^{\sqrt{2}}\frac{\sin\phi\;\text{d}\rho \text{d} \theta \text{d}\phi}
{\rho^2}$$
(use the quote button on this post to see how I did that ;) )

Please show your working.
 
  • Like
Likes 1 person
God, that looks complicated. I'll dedicate some time to actually learning how to write those integrals as nicely as you do after this week. I'm drowning in chemistry and math right now and haven't got a second to spare to learn itex right now :).

What I did was I drew a picture and then basically looked at it.
The limits for theta are obvious. The phi I got from setting x=0 on the upper limit and getting z=-x in the zy plane, this is 45 degree line, it's negative so 3pi / 4.

As for the rho limits, that's where I get less sure of what I'm doing.

I think that what I'm looking for is an ice cream cone-shape. Therefore the rho starts from 0 and ... Oh no wait I think I see it now. If I set the z-limits equal to each other I get the intercept which is r=1.
So then rho would go from 0 to 1. Correct?
 
Feodalherren said:

Homework Statement


coords.png



Homework Equations





The Attempt at a Solution


Is this the correct setup?

\int^{\pi}_{\frac{3\pi}{4}}\int^{2\pi}_{0}\int^{\sqrt{2}}_{0}\frac{1}{\rho^{2}} rho^2 Sin\phi d\rho d\theta d\phi

I gave up on itex. It was either that or my computer flying out the window. My apologies.

That is correct except why not cancel the ##\rho^2## factors?


Feodalherren said:
As for the rho limits, that's where I get less sure of what I'm doing.

I think that what I'm looking for is an ice cream cone-shape. Therefore the rho starts from 0 and ... Oh no wait I think I see it now. If I set the z-limits equal to each other I get the intercept which is r=1.
So then rho would go from 0 to 1. Correct?

No, ##\rho## isn't the same as ##r##. Your original limits are correct.
 
  • Like
Likes 1 person
Question: A clock's minute hand has length 4 and its hour hand has length 3. What is the distance between the tips at the moment when it is increasing most rapidly?(Putnam Exam Question) Answer: Making assumption that both the hands moves at constant angular velocities, the answer is ## \sqrt{7} .## But don't you think this assumption is somewhat doubtful and wrong?

Similar threads

Replies
3
Views
2K
  • · Replies 2 ·
Replies
2
Views
2K
  • · Replies 2 ·
Replies
2
Views
2K
  • · Replies 34 ·
2
Replies
34
Views
3K
  • · Replies 4 ·
Replies
4
Views
1K
  • · Replies 19 ·
Replies
19
Views
2K
  • · Replies 3 ·
Replies
3
Views
3K
Replies
14
Views
2K
  • · Replies 9 ·
Replies
9
Views
2K
Replies
3
Views
1K